LSAT and Law School Admissions Forum

Get expert LSAT preparation and law school admissions advice from PowerScore Test Preparation.

User avatar
 Dave Killoran
PowerScore Staff
  • PowerScore Staff
  • Posts: 5853
  • Joined: Mar 25, 2011
|
#26434
Complete Question Explanation
(The complete setup for this game can be found here: lsat/viewtopic.php?t=11009)

The correct answer choice is (B)

L, P, J, and N are the only names that could possibly be ranked among the three most popular. Therefore, four is the maximum possible number of soft drink names any one of which could be among the three most popular. It follows that answer choice (B) is correct.
 excitedforthelsat
  • Posts: 0
  • Joined: Jan 21, 2015
|
#18038
Hi,

There's a set of game problems on lesson 1 on page 1-25 under October 1992 Questions 7-12.

For question number 11, it asks what is the maximum possible number of the soft drink names any one of which could be among the three most popular?

According to the answer explanations, it says that L, P,J, and N are the only names that could possibly be ranked among the three most popular. Why would N be considered one of the three most popular?

Thank you for your help!

Sincerely,
Christine
 Emily Haney-Caron
PowerScore Staff
  • PowerScore Staff
  • Posts: 577
  • Joined: Jan 12, 2012
|
#18042
Hi Christine!

Thanks for the question. This is a game where having a solid set up is really critical, so that you have a clear sense of the sequencing relationships. Once I diagrammed the rules, I can see that L has to be more popular than P, and P has to be more popular than N and O (rule 5), and I know that two variables are less popular than O (K and M) and therefore can't be in the top 3. I also know that J is more popular than O (Rule 1), but I don't know how J might relate to L, P, or N. Therefore, L, P, and J will always come before O, K, and M. N isn't least popular, but could be third popular - we just don't know. Therefore, N can be in the top three, or not, and should be included in answer 11. Does that help?
 Adam Tyson
PowerScore Staff
  • PowerScore Staff
  • Posts: 5153
  • Joined: Apr 14, 2011
|
#18052
Looked at another way, how many variables MUST be in front of N? Only two - L and P. That means N can be as early as third.
 badarm
  • Posts: 16
  • Joined: Sep 28, 2016
|
#29178
Can you explain why L, J, N and P must be one of the soft drink names that could be in the top 3? I'm having a very hard time with this question. Thank You!!

-M
User avatar
 Jonathan Evans
PowerScore Staff
  • PowerScore Staff
  • Posts: 726
  • Joined: Jun 09, 2016
|
#29179
Hi, badarm,

Good question. The way this question is worded is somewhat hard to follow, but if you have difficulty understanding something, try parsing the question stem out part by part and explaining it to yourself in your own words. For instance, in this case:

"What is the maximum possible number of the soft drink names anyone of which could be among the three most popular?"

Start with the beginning:

"What is the maximum possible number of the soft drink names anyone of which"

Translate that:

"Which of the soft drinks"

Then:

"could be among the three most popular?"

So:

"could be in the top three?"

Rephrase for yourself:

"Which of the soft drinks could be in the top three?"

Now from your diagram, something that looks like this probably:

L
|
P J
/ \|
N O
\ |
| K
|/
M

You see that the only ones you can stretch out to be at the front are some combination of L, P, J, and N. The rest have to have at least three before them. So those are the four that could be in the top three. I hope this makes sense.
 Salli
  • Posts: 5
  • Joined: Mar 16, 2017
|
#33686
Do you have a diagram of this scenario?

Is there information that shows how Q's 8-12 came to their final answer?
 Robert Carroll
PowerScore Staff
  • PowerScore Staff
  • Posts: 1787
  • Joined: Dec 06, 2013
|
#33716
Hi Salli,

The diagram and explanations of the questions are found here:

..... lsat/viewforum.php?f=431

If you have further questions after reading that, just let us know and we'll work those out with you. Thanks!

Robert Carroll
 mattp72@gmail.com
  • Posts: 2
  • Joined: Jun 19, 2017
|
#36158
Please explain why it was not A, Thank you.
 Luke Haqq
PowerScore Staff
  • PowerScore Staff
  • Posts: 747
  • Joined: Apr 26, 2012
|
#36166
Hi mattp72,

Happy to try to help. So answer (A) states that there are 3 possibilities, (B) states there are 4, and the question is asking for the maximum number of variables that could be in the first 3 spots. The end of the previous post noted which variables could fit into those spots:
You see that the only ones you can stretch out to be at the front are some combination of L, P, J, and N. The rest have to have at least three before them. So those are the four that could be in the top three. I hope this makes sense.
Consider why L, P, J, and N are the only ones that could fit in the first 3 places. M certainly couldn't one of the first 3, since we know K, O, J, P, and L all have to come before M. We know it can't be K, because O, J, P, and L all have to come before K. And we know it can't be O, because J, P, and L all have to come before O--in other words, O could only be 4th at the earliest.

Since we started with 7 variables and eliminated M, K, and O, that means there are 4 possible variables that could be in the first 3 spots, which is why (B) rather than (A) is the right answer regarding the maximum number of variables.

Get the most out of your LSAT Prep Plus subscription.

Analyze and track your performance with our Testing and Analytics Package.